Croatian Mathematical Olympiad | 2005 Q11.1

แชร์
ฝัง
  • เผยแพร่เมื่อ 10 มิ.ย. 2024
  • We solve a nice number theory problem from the 2005 Croatian Mathematical Olympiad.
    Please Subscribe: th-cam.com/users/michaelpennma...
    Personal Website: www.michael-penn.net
    Randolph College Math: www.randolphcollege.edu/mathem...
    Randolph College Math and Science on Facebook: / randolph.science
    Research Gate profile: www.researchgate.net/profile/...
    Google Scholar profile: scholar.google.com/citations?...
    If you are going to use an ad-blocker, considering using brave and tipping me BAT!
    brave.com/sdp793
    Books I like:
    Abstract Algebra:
    Judson(online): abstract.ups.edu/
    Judson(print): amzn.to/2Xg92wD
    Dummit and Foote: amzn.to/2zYOrok
    Gallian: amzn.to/2zg4YEo
    Artin: amzn.to/2LQ8l7C
    Differential Forms:
    Bachman: amzn.to/2z9wljH
    Number Theory:
    Crisman(online): math.gordon.edu/ntic/
    Strayer: amzn.to/3bXwLah
    Andrews: amzn.to/2zWlOZ0
    Analysis:
    Abbot: amzn.to/3cwYtuF
    How to think about Analysis: amzn.to/2AIhwVm
    Calculus:
    OpenStax(online): openstax.org/subjects/math
    OpenStax Vol 1: amzn.to/2zlreN8
    OpenStax Vol 2: amzn.to/2TtwoxH
    OpenStax Vol 3: amzn.to/3bPJ3Bn

ความคิดเห็น • 117

  • @avivavital3769
    @avivavital3769 3 ปีที่แล้ว +132

    There are 2 little mistakes in the last board regarding the case that k=k.
    2. When regarding the case of k=2 in the last line, you didn't plug it in correctly to the right hand side, the equation should be:
    2l!=2+l!+m! => l!=m!+2, reducing that mod m, gives that 2 is congruent to 0 mod m which can only work for m=2, which is possible according to the first mistake.
    Plugging m=k=2 into the original equation gives:
    2l!=2+l!+2 => l!=4 but there is no solution here.

    • @demenion3521
      @demenion3521 3 ปีที่แล้ว +6

      the first one is not really a mistake as m=k would give the same kind of equation that he got for the case k=l before.

    • @demenion3521
      @demenion3521 3 ปีที่แล้ว +1

      @David Schmitz but you can still use the same argument. if m=k, then reducing the equation mod l+1 also gives either 0=2 or -1=2 mod l+1. neither of these give a new solution

    • @suryanshsharma2451
      @suryanshsharma2451 3 ปีที่แล้ว +1

      @@demenion3521 yaa

    • @antoniopalacios8160
      @antoniopalacios8160 3 ปีที่แล้ว +5

      @@demenion3521 actually, he would have to check the k=m

    • @alexey_burkov
      @alexey_burkov 3 ปีที่แล้ว +1

      @@antoniopalacios8160 I think you are not right a little. As soon as k
      2=0(mod m) ->
      m=2
      And k=2, k=m and it is possible on this stage. Now we must check the equation with this numbers, and find out that l! must be equal to 4...

  • @putinsgaytwin4272
    @putinsgaytwin4272 3 ปีที่แล้ว +21

    When you procrastinate maths homework to watch more complicated maths 😎

  • @adrianfrauca8118
    @adrianfrauca8118 3 ปีที่แล้ว +15

    The second part of Wilson's theorem has an extra case, which is n = p^2 for a prime p. That is proved by noting that 1 < p < 2p < p^2-1 (except for n = 2^2 = 4) and thus p^2 | 2p^2 | (p^2-1)!

  • @timurpryadilin8830
    @timurpryadilin8830 3 ปีที่แล้ว +35

    17:10, it should actually be 2l!=2+l!+m!, which gives l! =m! +2, but this is impossible

  • @hans-juergenbrasch3683
    @hans-juergenbrasch3683 3 ปีที่แล้ว +3

    No need to employ Wilson etc. By symmetry assume k=< l
    For k=k. If m>k, then all terms except the addend 1 are divisible by k+1>1, a contradiction. So m=k and l!=2+l!/k! requiring k>1. Similarly to before, this yields the contradiction k+1>2 divides the addend 2.
    Therefore, we must have k=l and get l!=2+m!/l! and therefore, l>2 and m>=l which yields m>l. Let d=min(m-l,l), then d! divides l! and m!/l! (note, that (m-l)! divides m!/l! using the binomial coefficient definition). So d! divides 2 and hence d=1 or d=2 smaller than l, so d=m-l.
    d=1 gives l!=3+l. Division by l>2 requires l=3 and this yields a solution k=l=3, m=4.
    d=2 gives l!=2+(l+1)(l+2)= l^2+3l+4. Division by l>2 yields l=4 which does not solve the equation.

  • @backyard282
    @backyard282 3 ปีที่แล้ว +30

    Croat here, thanks for featuring my country :)

    • @ajakowski
      @ajakowski 3 ปีที่แล้ว +2

      I am not a Croat, but I got annoyed when I saw Abramović misspelled :)

  • @goodplacetostop2973
    @goodplacetostop2973 3 ปีที่แล้ว +52

    18:30 and by the way, how would you say “That’s a good place to stop” in Croatian ?

    • @thapakaji8579
      @thapakaji8579 3 ปีที่แล้ว +5

      ok

    • @birdbeakbeardneck3617
      @birdbeakbeardneck3617 3 ปีที่แล้ว +4

      the vid got 9 likes u got 5

    • @thapakaji8579
      @thapakaji8579 3 ปีที่แล้ว +7

      @@birdbeakbeardneck3617 he does the work of god dude...

    • @marting5019
      @marting5019 3 ปีที่แล้ว +26

      "To je dobro mjesto za stati."
      greetings from Zagreb :)

  • @andrijaadamovic5129
    @andrijaadamovic5129 3 ปีที่แล้ว +5

    Very elegant solution. Btw that problem is from Croatian state competition 2005, not from the Croatian Olympiad 2005.

  • @Dubravko_Sabolic
    @Dubravko_Sabolic 3 ปีที่แล้ว +1

    Man, you are practically a Croat. And you make some very good videos... Greetings from Zagreb. :-)

  • @madcapprof
    @madcapprof 3 ปีที่แล้ว +13

    The condition of n=p^2 for some odd prime p, also needs to be solved in the Wilson's thrm corollary.

    • @timurpryadilin8830
      @timurpryadilin8830 3 ปีที่แล้ว +2

      if p>2, then p < p^2 and 2p< p^2, and thir multiplication gives 0 mod p^2

    • @madcapprof
      @madcapprof 3 ปีที่แล้ว +3

      @@timurpryadilin8830 My comment was a suggestion for including this case. I know the proof and wasn't asking a question.

    • @ittaloceara
      @ittaloceara 3 ปีที่แล้ว

      why.

    • @sammyboy7094
      @sammyboy7094 3 ปีที่แล้ว +3

      Because numbers of the form n=p^2 don’t have two divisors a,b such that 2

  • @Reidemeistermoves
    @Reidemeistermoves 3 ปีที่แล้ว +1

    Great video! I'd love to see a video where you talk about topics in your research area

    • @user-fh5rm2ef4n
      @user-fh5rm2ef4n 3 ปีที่แล้ว

      Actually there is such a video. I guess it was done by redpenblackpen. That was a stream

  • @rubyjha4298
    @rubyjha4298 3 ปีที่แล้ว

    Love your videos ❤

  • @jimschneider799
    @jimschneider799 3 ปีที่แล้ว

    As Aviv Avital pointed out, m is not necessarily strictly larger than k. Also, the equation that results from dividing through by l!, which is k! = 1 + k!/l! + m!/l!, implies that 1 < k! < 3, since 0 < k!/l!

  • @mislav9951
    @mislav9951 3 ปีที่แล้ว +21

    Im from Croatia! Its cool to know that youve been in my country lol

    • @harish6787
      @harish6787 3 ปีที่แล้ว +4

      Oh he is from Croatia awesome

    • @toljanginis2785
      @toljanginis2785 3 ปีที่แล้ว +5

      I'm from Dubrovnik, it's amazing that Michael visits us from time to time!

    • @sanelprtenjaca9776
      @sanelprtenjaca9776 3 ปีที่แล้ว +3

      Cheers from Rijeka

    • @matejcataric2259
      @matejcataric2259 3 ปีที่แล้ว +5

      Meni je drago da ima Hrvata na ovom kanalu ❤️

    • @toljanginis2785
      @toljanginis2785 3 ปีที่แล้ว +2

      @@matejcataric2259 Treba širiti i naše znanje😄

  • @hnnagarathna7286
    @hnnagarathna7286 3 ปีที่แล้ว

    Just an hour ago this is the fastest time evr that I've got ur video as recommendation

  • @hassanalihusseini1717
    @hassanalihusseini1717 3 ปีที่แล้ว

    Thank you for this number theory proof!

  • @suryanshsharma2451
    @suryanshsharma2451 3 ปีที่แล้ว +1

    Great teacher

  • @robertgerbicz
    @robertgerbicz 3 ปีที่แล้ว +6

    You really don't need Wilson's theorem to prove that
    k!=2+m!/k! has no solution (other than k=3,m=4).
    If k>=3 and m>=k+3 then it has no solution because the left side is divisible by 3,
    but the right side gives 2 as a residue mod 3.
    That gives us few cases to check: k

    • @Goku_is_my_idol
      @Goku_is_my_idol 3 ปีที่แล้ว +1

      Yes thats the way i did it too.
      k!=2+(k+1)(k+2) or k!=2+(k+1)
      This is for k>=3
      Next put mod3 for both sides
      In first case we gotta have (k+1)(k+2)=1(mod3) which is impossible
      And next k!=k+3 for k=3
      Which is the only solution
      We can easily check for k=1 and k=2 which gives us no solutions
      (I dont know wilsons theorem. If i had known maybe i would have used it. But this ques doesnt really require that)

    • @raghaviyer3097
      @raghaviyer3097 3 ปีที่แล้ว

      @@Goku_is_my_idol me too!

  • @lexyeevee
    @lexyeevee 3 ปีที่แล้ว +8

    4:35 this isn't possible if n is a square of a prime p, e.g. 9. the same idea does still work because p is necessarily greater than 2, so both p and 2p appear in (n - 1)! - which is the same reason 4 /doesn't/ work

    • @zeravam
      @zeravam 3 ปีที่แล้ว

      9 is not prime

    • @lexyeevee
      @lexyeevee 3 ปีที่แล้ว

      @@zeravam ...yes, it's the square of a prime

    • @agfd5659
      @agfd5659 3 ปีที่แล้ว

      Lol, like a minute after he had explained that proof, it clicked for me that he missed that and I decided to scroll into the comments to find yours.

  • @SlidellRobotics
    @SlidellRobotics 3 ปีที่แล้ว

    for k=l, m!/k! being whole means m>=k, not m>k. It turns out in both cases where he uses the fact it doesn't matter. The first was the k! l! < .. < k! l!, which was still proven. The other was for k=2, where m=1 is actually less than k.

  • @kasraafshar255
    @kasraafshar255 3 ปีที่แล้ว

    One of the best olympiad question and solutions channels but just one thing at the end of this video , considering k = 2 makes that k!=2 not k!=1 that you wrote on ths right side ; but not such a big mistake because it gives that 2l!= 2 + l! + m! , so we have l! = 2 + m! That for l>=3 there is no solution because u can take left and right side congerate to each other module 6 but RHS will never be congerate to 0 module 6 so l is less than 3 and in that case we also have not any solutions .
    From a high school student from iran who is in love with math 🙏🇮🇷

  • @udic01
    @udic01 3 ปีที่แล้ว +1

    13:57 m does not have to be strictly bigger than k. It could be equal to k (and the proof still stays the same)

  • @pokoknyaakuimut001
    @pokoknyaakuimut001 3 ปีที่แล้ว

    So much thanks from Indonesia 😇😇😇

  • @ribozyme2899
    @ribozyme2899 3 ปีที่แล้ว

    7:30 This is also possible without Wilson's theorem.
    Case m2k: m!>(2k)!>k!^2=2k!+m!, so 0>k!, which is a contradiction. (the only exception is m=1 and k=0, which isn't a solution)

  • @jvranstify
    @jvranstify 3 ปีที่แล้ว +3

    Writing this as K!L!=K!(1 + L!/K!) + M! implies that L >= K and similar argument for K >= L, so K = L

    • @ThePharphis
      @ThePharphis 3 ปีที่แล้ว +1

      oh I like this one. Seems a lot easier and quite intuitive.

    • @ThePharphis
      @ThePharphis 3 ปีที่แล้ว

      @@angelmendez-rivera351 On second thought I'm not so sure. In fact I don't think it does.

    • @jvranstify
      @jvranstify 3 ปีที่แล้ว

      @@angelmendez-rivera351 Because 1 + L! /K! then needs to be integer, etc...

    • @jvranstify
      @jvranstify 3 ปีที่แล้ว

      @@angelmendez-rivera351 must have been sleeping

  • @Pika250
    @Pika250 3 ปีที่แล้ว

    You forgot to address a case of Wilson's theorem, where if n = p^2 where p is an odd prime, then n divides p * 2p and both p and 2p are strictly between 1 and n.

  • @lucassandleris4486
    @lucassandleris4486 3 ปีที่แล้ว +3

    When solving the case k=l, it might be easier to just take everything mod3.
    If k

    • @Goku_is_my_idol
      @Goku_is_my_idol 3 ปีที่แล้ว

      Thats the way i did it too! (I dont know wilson theorem yet. Still a student)

    • @themathsgeek8528
      @themathsgeek8528 2 ปีที่แล้ว

      @@Goku_is_my_idol nice

  • @leif1075
    @leif1075 3 ปีที่แล้ว

    I'm curious Michael why you chose to specialize in that area or branch or field of math..representation theory and vertex operator algebra and not any other?

  • @arshadkhan6562
    @arshadkhan6562 3 ปีที่แล้ว

    exact same question came up in a British Mathematics Olympiad Paper!

  • @boubidebibou4547
    @boubidebibou4547 3 ปีที่แล้ว

    The most famous board in the world :D
    Mathematics are a way to dream... So thx Michael to keep us dreaming !

  • @donaldbiden7927
    @donaldbiden7927 3 ปีที่แล้ว +1

    My experience is that Number Theory proofs are the longest.

  • @user-A168
    @user-A168 3 ปีที่แล้ว +1

    Good

  • @protectionsecularism6090
    @protectionsecularism6090 3 ปีที่แล้ว

    Please topology and more advanced differential geometry

  • @digxx
    @digxx 3 ปีที่แล้ว

    Maybe it is trivial, but I don't see it: What about the case m=k if l>k? For then we would have l!=2+l!/m! = 2 + l*(l-1)*...*(m+1). Why doesn't have this a solution?

  • @mcwulf25
    @mcwulf25 3 ปีที่แล้ว

    How can you say m!/l!

  • @HagenvonEitzen
    @HagenvonEitzen 3 ปีที่แล้ว

    I think this is faster for the k 2, then m! =k! l! - k! - l! >= 6 l!-l!-l!=4 l!, so m > l, so k! = k! l! -l! -m! is a multiple of l!, which is absurd

  • @copperfield42
    @copperfield42 3 ปีที่แล้ว

    15:05 why it can't be that k!/l! + m!/l! -> (k! + m!)/l! result in a integer?

  • @jkid1134
    @jkid1134 3 ปีที่แล้ว

    I don't think your proof of wilson's theorem accounts for squares of primes, eg 9 cannot be decomposed into two distinct integers greater than 2.

  • @jomama3465
    @jomama3465 3 ปีที่แล้ว

    This channel derseves more subscribers.

  • @kozokosa9289
    @kozokosa9289 3 ปีที่แล้ว

    Didn't mindyourdecisions already cover this?

  • @adeolugboji3645
    @adeolugboji3645 3 ปีที่แล้ว

    I don’t understand the entire Wilson’s theorem part. He says (n-1)! is identical to -1 mod n, but I don’t see how that can ever be right.

    • @adeolugboji3645
      @adeolugboji3645 3 ปีที่แล้ว

      Diego Marra What is a multiplicative inverse? Apologies but I am a complete novice at group theory

  • @vigneshvenkataraman8
    @vigneshvenkataraman8 3 ปีที่แล้ว +1

    I don't know a lot of Croatian , but does anyone else think that he looks like Rakitic

  • @MonzennCarloMallari
    @MonzennCarloMallari 2 ปีที่แล้ว

    For a second I thought you just had to shout the letters

  • @user-mt9ux2di6u
    @user-mt9ux2di6u 3 ปีที่แล้ว +2

    I don't understand, what's the multiplicative inverse Modula n?

    • @lejamtelpaul9944
      @lejamtelpaul9944 3 ปีที่แล้ว +2

      Good question, I don’t get it too

    • @dariobarisic3502
      @dariobarisic3502 3 ปีที่แล้ว +8

      Multiplicative inverse of "x" (mod n) , let's denote it "y", is a number with property x*y = 1 (mod n). This is my guess based on his explanation of Wilson's theorem.
      For example, take mod 7. Inverse of zero is non existent, inverse of 1 is 1, inverse of 2 is 4 (because 2*4 =8 which is congruent to 1 mod 7), inverse of 3 is 5, and 6 is its own inverse. Same as in the video where he said for mod n, 1 and n-1 are their own inverses.

    • @3manthing
      @3manthing 3 ปีที่แล้ว +2

      When examining whole numbers modulo some number n, the set colapses only to 1,2,3,...,(n-2),(n-1). Every number bigger than n is being represented modulo n by one of these integers 1,....,(n-1). There is a special rule when adding and multiplying these numbers so when adding/multiplying two numbers modulo n, you still remain in the same set modulo n. Knowing this, multiplying different numbers modulo n might result in these two numbers resulting 1 when multipliyed. That means that the first number is multiplicative inverse modulo n of the other and vice versa.

    • @user-mt9ux2di6u
      @user-mt9ux2di6u 3 ปีที่แล้ว

      @@dariobarisic3502 thanks

    • @dariobarisic3502
      @dariobarisic3502 3 ปีที่แล้ว +2

      @@3manthing I'd also add that if we deal with (mod p) where "p" is prime, than inverses are unique, at least for all n0), so there is "x" which gives z=1.

  • @Tome281
    @Tome281 3 ปีที่แล้ว

    Drazen Adamovic is one of my professors, the world really is a tiny place

  • @aldues00
    @aldues00 3 ปีที่แล้ว

    what if a=b?

  • @eldattackkrossa9886
    @eldattackkrossa9886 3 ปีที่แล้ว

    let's go

  • @bsuperbrain
    @bsuperbrain 3 ปีที่แล้ว

    13:37 "we know that one is a whole number" - do we? Prove it! :D

    • @sudhansh6998
      @sudhansh6998 3 ปีที่แล้ว

      lol just use peano axioms

  • @throwawayuser9931
    @throwawayuser9931 3 ปีที่แล้ว +1

    Very nice prob solving in this channel actually.......
    Tho a lot of bashing involved

  • @bhanusri3732
    @bhanusri3732 3 ปีที่แล้ว

    17:45 l>m doesn't always mean m divides l??

    • @bhanusri3732
      @bhanusri3732 3 ปีที่แล้ว

      @@angelmendez-rivera351 He said that m divides l because l>m but its not always the case for example 5>2

    • @bhanusri3732
      @bhanusri3732 3 ปีที่แล้ว

      @@angelmendez-rivera351 Ohh thanks a lot. I misunderstood it

  • @gasparrobert9769
    @gasparrobert9769 3 ปีที่แล้ว

    What the hell is mod aha ?

  • @anggalol
    @anggalol 3 ปีที่แล้ว

    You should make patreon

  • @Macisordi
    @Macisordi 3 ปีที่แล้ว

    Ragusa Is beautiful

  • @PachinkoTendo
    @PachinkoTendo 3 ปีที่แล้ว

    Didn't expect this. Wilson's theorem is overkill, you can solve this with basic arithmetic and using an even/odd parity argument.

    • @leif1075
      @leif1075 3 ปีที่แล้ว

      Right did you just kind of plug and chug guessing starting with small integers?

  • @thephysicistcuber175
    @thephysicistcuber175 3 ปีที่แล้ว

    You technically forgot to consider the case k=l=0, but that trivially yields no solutions.
    EDIT: Misleading thumbnail.

    • @vaxjoaberg9452
      @vaxjoaberg9452 3 ปีที่แล้ว +4

      It's been a long time since I took a math class but I'm pretty sure 0 isn't a positive integer. :P

    • @thephysicistcuber175
      @thephysicistcuber175 3 ปีที่แล้ว

      @@vaxjoaberg9452 Oh sh!t. Misleading thumbnail.

    • @nathanisbored
      @nathanisbored 3 ปีที่แล้ว +1

      @@thephysicistcuber175 unless he since changed the thumbnail, i dont see what's misleading about it. the set N conventionally does not include 0

    • @thephysicistcuber175
      @thephysicistcuber175 3 ปีที่แล้ว +1

      @@nathanisbored uhm no? 0 is conventionally natural.

    • @nathanisbored
      @nathanisbored 3 ปีที่แล้ว

      @@thephysicistcuber175 ok so i looked it up and apparently both conventions are commonly used. the way i learned it was
      N = {1,2,3,...}
      W = {0,1,2,3,...}
      Z = {...,-3,-2,-1,0,1,2,3,...}
      and if you want negatives only, i guess you would do:
      Z- = {-1,-2,-3,...}
      and Z+ would just be equal to N
      it seems in some conventions, Z = W, which is even more confusing. But anyway, i think its safe to assume in the thumbnail (and most of his videos), he was using the convention that N = {1,2,3,...}

  • @shikhapandey7423
    @shikhapandey7423 3 ปีที่แล้ว

    First